Difference between revisions of "2016 AMC 10A Problems/Problem 14"
(→Solution) |
(→Solution) |
||
Line 3: | Line 3: | ||
The amount of twos in our sum ranges from <math>0</math> to <math>1008</math>, with differences of <math>3</math> because <math>2 \cdot 3 = lcm(2, 3)</math>. | The amount of twos in our sum ranges from <math>0</math> to <math>1008</math>, with differences of <math>3</math> because <math>2 \cdot 3 = lcm(2, 3)</math>. | ||
− | The possible amount of twos is <math>\frac{1008 - 0}{ | + | The possible amount of twos is <math>\frac{1008 - 0}{3} + 1 \Rightarrow \boxed{\textbf{(C)} 337}</math> |
Revision as of 17:59, 3 February 2016
Solution
The amount of twos in our sum ranges from to , with differences of because .
The possible amount of twos is